Skip to content

Commit 3798956

Browse files
committed
Update the typeset of Wick contractions.
1 parent 12cf6fb commit 3798956

File tree

4 files changed

+55
-14
lines changed

4 files changed

+55
-14
lines changed

[email protected]

10.2 MB
Binary file not shown.

[email protected]

Lines changed: 3 additions & 0 deletions
Original file line numberDiff line numberDiff line change
@@ -5,6 +5,7 @@
55
\setlength \feynhandblobsize {10mm}
66
\usepackage[natbib, style = phys, biblabel = brackets, sorting = none]{biblatex}
77
\addbibresource{reference.bib}
8+
\usepackage{zref-clever, simpler-wick}
89

910
\coverset{
1011
title = Quantum Many-Body Theory,
@@ -55,6 +56,8 @@
5556
\newweek
5657
\include{./context/weeka_[2025-11-11]}
5758
\include{./context/10_.tex}
59+
\newweek
60+
\include{./context/weekb_[2025-11-18]}
5861

5962
\printbibliography[title = {\refname\label{chap:bibliography}}]
6063

context/10.tex renamed to context/10_.tex

Lines changed: 14 additions & 14 deletions
Original file line numberDiff line numberDiff line change
@@ -107,7 +107,7 @@ \section{Contraction}
107107

108108
Define the contraction of two operators
109109
\begin{equation}
110-
\underbrace{A(t) B(t')}
110+
\wick{\c A(t) \c B(t')}
111111
= T_\epsilon(A(t) B(t')) - \mathcal N(A(t)B(t'))
112112
\end{equation}
113113
The operator identity
@@ -119,7 +119,7 @@ \section{Operator level of ``contraction''}
119119
\begin{example}[Two operators]
120120
We just look at the RHS of the contraction
121121
\[
122-
\underbrace{\gamma(t) \gamma^\dagger(')}
122+
\wick{\c \gamma(t) \c \gamma^\dagger(')}
123123
= T_\epsilon(\gamma(t) \gamma^\dagger(t')) - N(\gamma(t) \gamma^\dagger(t'))
124124
\]
125125
On the operator level. Where
@@ -131,7 +131,7 @@ \section{Operator level of ``contraction''}
131131
\]
132132
Then, we have to define the actual contraction
133133
\[
134-
\braket<\eta_0|\underbrace{\gamma \gamma^\dagger}|\eta_0>
134+
\braket<\eta_0|\wick{\c \gamma \c \gamma^\dagger}|\eta_0>
135135
= \iu G^{0,c}[\gamma, \gamma^\dagger]
136136
\]
137137
\end{example}
@@ -147,7 +147,7 @@ \section{Operator level of ``contraction''}
147147
1234 + 2134 + \cdots
148148
= (12 + 21)34 + T_\epsilon (12)(34)
149149
\xlongequal{\text{Contraction tricks}}
150-
(\overbrace{12} + N(12)) (34)
150+
(\wick{\c11\c12} + N(12)) (34)
151151
\]
152152
So, the observable
153153
\[
@@ -160,9 +160,9 @@ \section{Operator level of ``contraction''}
160160
1243 + 2143 \to \iu g_{12}(43), \quad
161161
\]
162162
The combines to $\iu g_{12}$ and $\iu g_{34}$,
163-
i.e., $T_\epsilon(\overbrace{12}\overbrace{34})$.
163+
i.e., $T_\epsilon(\wick{\c1 1\c1 2\c1 3\c1 4})$.
164164
Also for $1324 + 3124$ and $1342 + 3142$, which can be combined into
165-
$T_\epsilon(1234)$. (overbrace 13 and 24).
165+
$T_\epsilon(\wick{\c1 1\c2 2\c1 3\c2 4})$.
166166
The LHS have $4!$ terms in total, and we will have $4\times3/2 = 6$ contractions
167167
\[
168168
g_{12} g_{34} \quad g_{13} g_{24} \quad g_{14} g_{23}
@@ -171,10 +171,10 @@ \section{Operator level of ``contraction''}
171171
\begin{multline}
172172
T_\epsilon(123 \cdots 2N)
173173
= N(12\cdots 2N) + \binom{N\text{-product}}{\text{with ONE contraction}}
174-
N(\underbrace{12}34 \cdots)
175-
+ N(\underbrace{13}24 \cdots 2N) \\
174+
N(\wick{\c11\c12}34 \cdots)
175+
+ N(\wick{\c11\c13}24 \cdots 2N) \\
176176
+ \binom{N\text{-product}}{\text{with TWO contraction}}
177-
+ N(\underbrace{12}\underbrace{34}56 \cdots 2N) + \cdots
177+
+ N(\wick{\c11\c12\c13\c14}56 \cdots 2N) + \cdots
178178
+ \{\text{total contractions}\}
179179
\end{multline}
180180
\begin{example}[$T_\epsilon(123456)$]
@@ -188,9 +188,9 @@ \section{Operator level of ``contraction''}
188188
\end{example}
189189
Do the induction $2n$/$2n + 2$-Wick's
190190
\[
191-
(\underbrace{12}(34) + N(12)(34)) \cdot
192-
(\underbrace{56} + N(56)) \to
193-
\prod_{1, \cdots, 1}^n (\underbrace{i_1 i_2} + N(\\quad))
191+
(\wick{\c11\c12}(34) + N(12)(34)) \cdot
192+
(\wick{\c15\c16} + N(56)) \to
193+
\prod_{1, \cdots, 1}^n (\wick{\c i_1 \c i_2} + N(\quad))
194194
\]
195195
Together, we have $n$-terms
196196
\[
@@ -210,7 +210,7 @@ \section{Operator level of ``contraction''}
210210
The canonical algebra
211211
\[
212212
\{c_i, c_j^\dagger\} = \delta_{ij}, \quad
213-
\underbrace{\gamma \gamma^\dagger} = T_\epsilon - N(\cdots)
213+
\wick{\c\gamma \c\gamma^\dagger} = T_\epsilon - N(\cdots)
214214
\to s^2 \Tr(s^+, s^-)
215215
\]
216216
One is canonical algebra, and one is the Gaussion states.
@@ -406,7 +406,7 @@ \section{Unlabeled Feyman diagram}
406406
COmbines them Topo ineq.
407407
\[
408408
G_\Gamma \to S, \quad
409-
\cancel{\frac1{2^nn!m!}} \sum\{n||\text{contractionss}\} = \frac1s (\cdots)
409+
\cancel{\frac1{2^nn!m!}} \sum\{n\|\text{contraction}\} = \frac1s (\cdots)
410410
\]
411411
which cancels by ``All poss value-inv-transf'', for $v = \frac12$.
412412
\end{example}

context/weekb_[2025-11-18].tex

Lines changed: 38 additions & 0 deletions
Original file line numberDiff line numberDiff line change
@@ -0,0 +1,38 @@
1+
% !TeX root = ../main.tex
2+
3+
\section*{Homework \#11 [2025-11-18]}
4+
5+
\begin{problem}[label = prob.11.1]\leavevmode
6+
\begin{enumext}
7+
\item Write the time-ordered product
8+
\[
9+
T_\epsilon \{a_{k\sigma}(t_1)a_{l\sigma'}^\dagger(t_2)
10+
a_{m\sigma}(t_3) a_{n\sigma'}^\dagger(t)3\}
11+
\]
12+
in terms of normal products and suitable contractions.
13+
\item Express the expectation value of the time-ordered product in part
14+
(a) in the ground state $\ket|\eta_0>$ of the unperturbed system in terms
15+
of products of the free causal Green's functions.
16+
\end{enumext}
17+
\end{problem}
18+
\begin{solution}
19+
20+
\end{solution}
21+
22+
\begin{problem}
23+
Evaluate explicitly the expectation value of the time-ordered product in
24+
part 1 of \Autoref{prob.11.1} in the ground state $\eta|\eta_0>$ of the
25+
unperturbed system for the special case $k = l = m = n$, $\sigma = \sigma'$
26+
for
27+
\begin{enumext}[columns = 2]
28+
\item $t_1 > t_2 > t_3$,
29+
\item $t_1 > t_3 > t_2$.
30+
\end{enumext}
31+
Check the results by direct calculation of the expectation values, i.e.,
32+
without using Wick's theorem.
33+
\end{problem}
34+
\begin{solution}\leavevmode
35+
\begin{enumext}
36+
\item
37+
\end{enumext}
38+
\end{solution}

0 commit comments

Comments
 (0)